Torts Multiple Choice

अब Quizwiz के साथ अपने होमवर्क और परीक्षाओं को एस करें!

A pointed a gun at B scaring B and without authority or justification, A ordered B into a room and locked the door for 2 hours. B was awake the whole time and worried about what would happen until A released him unharmed. When A initially pointed gun at B, B was standing next to C and C dove for cover thinking the gun was pointed at her. In diving for cover C collided violently with D causing D to spill coffee on E's computer. E subsequently had to pay to have the computer repaired. Who may maintain an action against A?

B, C, D and E may all maintain actions against A

Store security guard sees person A stealing a watch by placing the watch in their coat on the store's security camera. Security guard immediately leaves her station to question A. When security guard gets to the floor of the store security guard accidentally detains B instead of A. B and A were dressed in the same coat and were about the same physical stature. As soon as the security guard begins to question B it becomes apparent that he detained the wrong person and B is released and free to go. Assuming the security guard made a reasonable mistake what result if B sues the store for false imprisonment?

A B will prevail unless he was detained in a manner that was reasonable to detain A. Correct. A merchant is privileged to detain an individual suspected of shoplifting. The fact that B was stopped instead of A due to the security guard's reasonable mistake does not affect the privilege. Reasonable mistake in exercising a privilege does not negate the privilege. So B would prevail unless B's detention would have been a reasonable one if A was detained.

City is 100 square miles and contains almost 60,000 trees along its streets. Barney is driving on a street during a windstorm when a tree on City property adjacent to the street falls on his car. Barney's car is damaged, and he suffers physical injuries. City had planted the tree some 15 years previously, and upon visual inspection by City workers, the tree appeared to be quite healthy. City has trained staff, which inspects each city tree annually, and employees of other City departments are under strict instruction to keep their eyes open for trees in a hazardous condition as they perform their regular duties. However, at the time of the storm, the tree was rotting due to rare disease from its inside out, and the weakened state of this particular tree caused it to fall under the pressure of the storm. Even when a tree does go bad as this one did, the chances of it suddenly falling in circumstances likely to produce serious property or personal injury are quite small. City had no actual knowledge of this tree's condition. Only a much more costly inspection program could have successfully detected the rotting tree that fell on Barney. In a negligence action brought by Barney against City, will Barney likely be able to hold city liable in negligence?

A Barney will likely not be able to hold City liable in negligence because the cost to City of detecting the dangerous condition of the tree was prohibitive. Correct. We know from the facts provided that only a "much more costly inspection program could have successfully detected the rotting tree" and that the chance of a diseased tree falling and causing injury to person or property is "quite small." Therefore, guarding against the small chance of harm was cost prohibitive, and City's failure to do so is not unreasonable.

Driver negligently runs a red light and hits Bob's daughter who suffers serious injury. Bob sees the accident from the sidewalk and suffers serious emotional harm. Bob sues Driver in negligence for his emotional injuries. Can Bob recover from Driver?

A Bob can recover from Driver because Driver negligently caused a sudden serious bodily injury to Bob's daughter and Bob perceived the event contemporaneously. Correct. A defendant who negligently inflicts sudden serious bodily injury to another is subject to liability for serious emotional harm caused to a person who contemporaneously perceived the accident and who is a close family member of the injured person. Restatement (Third) Torts § 48

Chair Palace manufactures various types of chairs, including an oak chair having five horizontal wooden bars that ergonomically cradle the human back. College Student has one of these oak chairs in his dorm room. Needing a book on a shelf out of his reach, College Student stands on the top bar of the chair in order to reach the book. The chair tips, and College Student falls, suffering serious harm. In a products liability lawsuit brought by College Student against Chair Palace, College Student alleges that the chair is defectively designed because it should have had the stability to support him while standing on its top bar or it should have had a differently designed back so that he could not have stood on the bar. Is Chair Place likely to be held liable?

A Chair Palace will likely not be held liable based upon a defective design of the chair because College Student's misuse of the product is so unreasonable that the risks it entails need not be designed against. Correct. An unreasonable misuse of a product will not subject the manufacturer to liability. See Restatement (Third) Torts: Products Liability § 2 and illustration 20 upon which this fact pattern is based.

Drug Co. negligently manufactures and sells a defective contraceptive pill, which fails to prevent Yolanda's pregnancy. Yolanda gives birth to Child A, who grows up and gives birth to Child B. Child B grows up to be a motorcycle enthusiast, who negligently causes a motorcycle accident that injures Charles. Charles seeks to recover from Drug Co. in a negligence action, alleging that Drug Co.'s negligence was a factual cause of Charles's injury. Will Charles's negligence action likely succeed or fail?

A Charles is correct that Drug Co. is a factual cause of Charles's injury, but his negligence action will likely fail for lack of proximate cause. Correct. Even though Drug Co. is a factual cause of Charles's injuries, his injuries are likely too remote from Drug Company's negligence in order for its negligence to be a proximate cause of his injuries. This question asks about proximate cause determinations in a products liability action grounded in negligence.

A state statute requires that the operator of a truck that becomes disabled on a highway promptly put out a warning sign at least 100 feet behind the truck. When a deflated tire disables Chuck's truck, he promptly places out a warning sign right next to the truck rather than at the 100-foot distance. Alice, approaching Chuck's truck from behind, does not see Chuck's warning sign until it is too late for her to stop. Her car strikes the rear of Chuck's truck, and she is injured in the collision. Alice sues Chuck in negligence, and the evidence at trial shows that Alice would have been able to stop in time had the warning sign been set at the 100-foot distance as required by statute. What effect, if any, does Chuck's violation of the statute have on his liability for negligence?

A Chuck's violation of the statute may be used to establish his negligence, because the basic purpose of the statute is to prevent accidents of this type and Alice is in the group of persons the statute is designed to protect. Correct. This response correctly provides the standard for using a statute to establish the standard of care in negligence. Restatement (Third) Torts § 14. This question asks about using a statute's violation to establish the standard of care in a negligence action.

Father was severely beaten by robber and is bringing a claim for assault and battery against robber. Unknown to robber, daughter witnessed father being beaten and suffered emotional distress as a result. If daughter brings a claim for intentional infliction of emotional distress (IIED) against robber, may daughter recover?

A Daughter may not recover because robber did not know she witnessed the father's beating. Correct. The rule by which bystanders may recover pursuant to IIED for tortious conduct directed at another is that the tortfeasor must have known or have been substantially certain the bystander witnessed the tortious conduct directed at another person. The reason for this is that IIED is not one of the torts within the original writ of trespass and so the intent does not transfer from the tortious conduct against the father so as to support the intent prong of the daughter's IIED claim in this scenario. In order for transferred intent to apply the tort intended and the tort committed must be within the original writ of trespass (Trespass to Land, Trespass to Chattel, False Imprisonment, Assault and Battery).

Defendant, driving a car, approaches Plaintiff's car, and properly steps on the brakes. For reasons unrelated to any negligence on Defendant's part, her brakes fail to function. Much to her surprise, Defendant's car continues forward, so she quickly pumps the brakes again, but to no avail. Defendant's car careens into Plaintiff's car, injuring Plaintiff. Upon reflecting calmly on Defendant's brake-failing situation, a better choice for Defendant would have been for her to turn her car quickly to the right. In Plaintiff's lawsuit against Defendant in which he seeks to recover from her in negligence, will Defendant likely be held liable in negligence?

A Defendant will likely not be held liable for negligence because in light of her emergency circumstances, she behaved as a reasonable prudent person. Correct. This question asks about the standard of care in a negligence action when the defendant is acting in an emergency, which requires a person to act reasonably in the emergency situation.

Doctor while conducting an operation on plaintiff's left eye decides to examine the rest of plaintiff's face and discovers that plaintiff has a deviated nasal septum. At the time of the discovery the plaintiff was under the effect of general anesthesia and unconscious. The doctor decides to correct the deviated septum while the plaintiff rather than do it separately which would require another operation. Which of the following is true?

A Doctor likely committed a battery. Correct. This is a classic case of battery in tort law. Battery is a dignitary tort and is based on the individual's right to permit or not allow contact with his or her body. Even though the doctor acted with "good" intentions, motive is not the same as intent. Here the doctor committed nonconsensual contact to the person of another because presumably he only had consent to operate on the patient's eye and perform any contact associated with that. If a reasonable person would find the extra contact to be harmful or offensive then battery would be actionable. Even though the damage may be minimal many intentional tort suits survive on nominal damages. The purpose of this is that the decisions become judicial statements on what is permitted and not permitted under the law.

Rosie and Vincent do not know each other and were camping independently in a heavily forested campground. Each one had a campfire, and each one negligently failed to ensure that the fire was extinguished upon retiring for the night. Due to unusually dry forest conditions and a stiff wind, both campfires escaped their sites. The two fires, burning out of control, eventually joined together and engulfed Company's hunting lodge, destroying it. Either fire alone would have destroyed the lodge. In a lawsuit brought by Company against both Rosie and Vincent alleging negligence, which of the following statements is correct regarding factual causation?

A Each of Rosie's and Vincent's negligence is a factual cause of the destruction of Company's hunting lodge. Correct. When multiple acts occur and each alone would have been a factual cause in the absence of the other act, each act is a factual cause of the harm. This is a question about factual causation when two independent acts of negligence join together to create one harm where either independent act alone would have been sufficient in creating the harm. Restatement (Third) Torts § 27.

Gas Co. supplies natural gas for the county and hires Supply Inc. to install a service line to carry gas to a new neighborhood. Supply Inc.'s employees negligently cause a slight leak in one of the gas pipes. The resulting leak causes an explosion that injures Neighbor. In a lawsuit brought by Neighbor against Gas Co., may Gas Co. be subject to vicarious or direct liability?

A Gas Co. may be subject to vicarious liability for the negligence of Supply Inc. because Gas Co. hired Supply Inc. to carry out an activity that posed a highly dangerous risk and Supply Inc.'s negligence was a factual cause of Neighbor's harm. Correct. This is a correct statement of the law regarding the vicarious responsibility of one who hires an independent contractor to carry out an activity that poses a highly dangerous risk and whose negligence in doing so causes injury to the plaintiff. Restatement (Third) Torts § 58.

Charles is a guest in Heights Hotel. The bathroom in the hotel includes a shower, which is protected by a sliding door made of ordinary glass. While taking a shower, Charles trips and falls on the glass door, causing it to shatter. The shards of glass cut Charles causing serious injuries. It is a standard practice among hotels to use shatterproof, safety glass rather than ordinary glass at shower enclosures. In a negligence action brought by Charles against Heights Hotel, what effect, if any, will Hotel's departure from the industry custom have on Hotel's negligence?

A Heights Hotel's departure from this custom is evidence of its negligence. Correct. A Defendant's departure from custom is evidence that the defendant was negligent, but such departure does not require a finding of negligence. Restatement (Third) § 13.

Hunter finishes his day working in the field and stops at a friend's house on his walk home from work. His friend's nine-year-old daughter, Katie, greets Hunter, who hands his loaded shotgun to her as he enters the house. The shotgun is neither especially heavy nor unwieldy. Nonetheless, Katie drops the shotgun, which lands on her toe, breaking it. In a negligence action against Hunter for Katie's broken toe, may Hunter be held liable in negligence?

A Hunter may not be held liable in negligence for Katie's broken toe because the risk that makes Hunter negligent is that Katie might shoot something or some person with the gun, not that she would drop it on her toe and hurt herself. Correct. This question asks about proximate causation determinations. Generally speaking, a defendant will be liable in negligence for those harms that result from the risks that make the actor's conduct tortious. See Restatement (Third) Torts § 29 and illustration 3, upon which this fact pattern is based. A broken toe is not what makes giving a child a rifle risky.

Private Citizen is walking through the park and hears two "average people" having a conversation, in which one is telling the other that her brother has had an affair with a well-known federal Senator and that the Senator's spouse is unaware of the affair. Private Citizen posts "Senator (name) is having an affair and spouse is unaware," on his Facebook page and after a day has received in excess of 1 million likes. Senator sues Private Citizen for defamation. Which of the following is a valid defense for the Private Citizen in responding to the suit?

A Private Citizen did not investigate to see whether the content of the conversation was true or false before posting. Correct. When the plaintiff in a defamation action is a public figure or public official the plaintiff must prove that the statement was published with actual malice. This means that there must be proof that the defendant knew of the falsity or recklessly disregarded the truth of the statement when they published the statement. The case law is clear that failure to investigate alone is not enough to prove actual malice even though it may be negligent to do so. The level of fault that a public official must prove is higher than negligence and is either recklessness or intentionally saying something she knows to be false.

Shopper insulted store clerk by saying that the store clerk had a disgusting body odor. Shopper intended to insult and embarrass store clerk and said it loud enough for the fellow shoppers in the vicinity to hear. Store clerk was mortified when other shoppers laughed. As a result of the insult, store clerk fainted and hit his head on a display and suffered a concussion. Assume that the shopper intentionally, impermissibly and offensively touched the store clerk while he made the comment but the store clerk was only mildly embarrassed and suffered no additional physical injuries due to the touch. Will store clerk be able to recover?

A Store clerk may be able to recover for both the impermissible contact and the mild embarrassment. Correct. The intentional, impermissible and offensive touching will constitute a battery. And the tort of battery unlike the tort of IIED is actionable on only nominal damage as it is a dignitary tort . The requirement of outrageous and extreme conduct and severe harm are only conditions precedent to recovery if the cause of action is Intentional Infliction of Emotional Distress. Here the store clerk will likely have an actionable battery and if the store clerk elects to sue in battery then any and all associated emotional harms are recoverable. It is only when the cause of action is IIED that the requirements of outrageous conduct and severe harm must be satisfied before recovery of emotional harm is permitted.

George, a concessionaire at a sports stadium, negligently collided with Fanny, a fan attending the game, knocking her to the ground. Fanny had an asymptomatic herniated disc in her back that results in her suffering serious back injury and pain as a result of George's collision with her. In a negligence action brought by Fanny against George, may George be subject to liability for Fanny's injuries?

A That Fanny could be injured was foreseeable from George's negligence and therefore George can be subject to liability. Correct. Pursuant to the eggshell skull plaintiff rule, a plaintiff's physical injuries are foreseeable.

Jeremy purchased a bottle of liquor from Liquor Barn. The liquor was bottled by Bottlers Inc., in bottles manufactured by A#1 Glass Co. While Jeremy was opening the bottle, it suddenly exploded, causing disfiguring cuts to his face. The fact finder determines that, originating with A#1 Glass Co., the bottle contained a weakness in the glass structure that caused Jeremy's harm and was a departure from the product's intended design. The weakness in the bottle's structure caused the bottle to explode. In a lawsuit brought by Jeremy against Liquor Barn, A#1 Glass Co., and Bottlers Inc., what type of defect did the bottle have?

A The bottle that caused Jeremy's harm contained a manufacturing defect. Correct. A manufacturing defect is a departure from a product's intended design. Restatement (Third) Torts: Products Liability § 2.

A state statute fixes a speed limit of 55 miles per hour for a state highway. Driver is driving her car on that highway at a speed of 50 miles per hour. Teenager unexpectedly darts from the sidewalk onto the highway. Despite a proper lookout, Driver is unable to stop in time and strikes Teenager, injuring him. Had Driver been driving at 40 miles per hour, she would have been able to avoid hitting Teenager. In his lawsuit, Teenager alleges that Driver was negligent in driving at a speed faster than 40 miles per hour. At the time the weather had been fine, the road and traffic conditions had been entirely ordinary, and no other circumstances suggested any special danger. Driver asks the court to rule as a matter of law that she did not breach the standard of care. Which statement regarding the court's ruling on the standard of care and Driver's compliance with the statute is correct?

A The court would be justified in ruling as a matter of law that Driver did not breach the standard of care by driving at the speed she was driving at the time of the accident due to the favorable weather and road conditions at the time of the accident. Correct. Although usually compliance with a statute may be used merely as evidence of non-negligence, the court may rule as a matter of law that compliance amounts to non-negligence, such as in the facts of this question when the weather was fine, road conditions are ordinary, and there was nothing to suggest any special dangers.

Toddler, a two-year-old child suffering from a bacterial infection, received a routine childhood vaccination with a vaccine manufactured by Vaco. Shortly after receiving the vaccine, Toddler spiked a very high fever, went into respiratory arrest, and died. Toddler's estate sues Vaco in negligence. At trial, Toddler's estate provides competent evidence that Toddler died due to a combination of the infection and the vaccine and, if either toddler had not had an infection or if Toddler had not been vaccinated, death would not have ensued. Regarding the factual causation element of the negligence action, has Toddler's estate satisfied its burden of production to prove that the vaccine was a factual cause of Toddler's death?

A Toddler's estate has satisfied its burden of production to show that the vaccine was a factual cause of Toddler's death. Correct. See Restatement (Third) Torts § 26 and illustration 4, upon which this problem is based. The estate's evidence that Toddler died due to a combination of the infection and the vaccine coupled with proof that either toddler had not had an infection or if Toddler had not been vaccinated, death would not have ensued, is sufficient to prove factual causation.

Joe has just finished eating his lunch and is walking alone down the sidewalk in front of a restaurant when he suddenly suffers a severe asthma attack. Waiter, who works at the restaurant but is on a break at the time, sees Joe through a window and realizes that Joe is suffering an asthma attack. Nonetheless, Waiter chooses to do nothing to help Joe. As a result, the appropriate medical care for Joe is delayed, causing Joe serious physical injuries that could have otherwise been avoided. Joe sues Waiter in negligence for his injuries that result from the delay in his receiving medical care. May Waiter be liable to Joe in negligence?

A Waiter may not be subject to liability in negligence because Waiter had no affirmative duty to render aid or call for assistance for Joe. Correct. Here, Waiter did not create any risk of harm to Joe. This question asks about one's duty to act affirmatively in negligence. The general rule is that a defendant has no affirmative duty to act unless the defendant has created a risk of physical or emotional harm to another. Restatement (Third) Torts § 37.

Ticky Tack Co. manufactures a chemical glue used in do-it-yourself home improvement projects. Susie purchased the glue from Home Store to lay faux wood in her kitchen, which she has been remodeling. In large, capital letters on the front of the container of glue was a warning stating that the fumes of the glue were highly flammable and toxic and that the glue should only be used in a place with adequate ventilation. The box warnings also provided that all sources of fire should be extinguished. Susie opened the windows in her kitchen, but failed to extinguish the pilot light in her gas stove. Midway through laying the tile, the pilot light suddenly ignited the fumes from the glue, and Susie was seriously burned as a result. In a products liability action against Ticky Tack, Susie contends that the warnings were inadequate in failing to state specifically that gas stove pilot lights should be extinguished. Which statement regarding the adequacy of the warnings in a products liability action is correct?

A Whether the warnings were inadequate is a question of fact. Correct. Whether a product is defective based on inadequate warnings is a question of fact.

Drug Manufacturer along with many other drug companies manufactured, produced, and marketed a medication, DRG, designed to prevent miscarriages in pregnancy. The drug was administered to pregnant women. Unfortunately, women who ingested DRG, thereby exposing their unborn daughters to DRG, gave birth to daughters who developed cancerous vaginal and cervical growths. The unique type of cancer these daughters developed was a fast-spreading and deadly disease requiring radical surgery to prevent its spread. Daughter's mother ingested DRG, exposing Daughter to the drug prior to her birth. The evidence also shows that this unique type of cancer is most probably caused by exposure to DRG. Daughter has developed the cancerous vaginal and cervical growths, which are only developed by women like her whose moms ingested DRG. Daughter has sued Drug Manufacturer, seeking to hold it responsible in negligence for its manufacture of DRG and Daughter's resultant injuries. The evidence at trial establishes that Drug Manufacturer produced DRG at the time that Daughter's mother was prescribed and took the drug. However, Daughter is unable to prove with certainty that the DRG that her mother ingested was manufactured by Drug Manufacturer and not one of the other several manufacturers of the drug at the time the drug was prescribed to her mom. At best, the evidence suggests that Drug Manufacturer possibly manufactured the DRG Daughter's mom ingested. Drug Manufacturer has moved for summary judgment, arguing that Daughter's cause of action against it should be dismissed for failing to establish factual causation. Should the court grant Drug Manufacturer's motion for summary judgment?

A Yes, evidence that Drug Manufacturer manufactured the drug to which Plaintiff was exposed is insufficient to establish factual case. Correct. The evidence that Drug Manufacturer possibly manufactured the DRG pill is insufficient proof of factual causation, which requires proof that the defendant's negligence more likely than not (or probably) brought about the harm to Plaintiff.

Traditionally, contributory negligence as a defense to a negligence action:

A was a complete bar to the plaintiff's recovery. Correct. It is a correct statement of the traditional defense of contributory negligence.

Alice sues Bill alleging negligence. Bill defends, claiming that Alice's negligence also contributed to her injuries. At trial, the factfinder assigns 50% responsibility to Alice and 50% responsibility to Bill. The factfinder also finds that Alice's damages are $100,000. In a modified comparative fault (51% or "not greater than") jurisdiction, what amount, if any, is Alice entitled to recover?

Alice is entitled to recover $50,000 from Bill because her recoverable damages are her damages reduced by the amount of fault attributable to her. Correct. In a modified not greater than jurisdiction, the plaintiff is not completely barred from recovery unless her fault is greater than that of the defendant. Here, plaintiff's fault is not greater than that of the defendant, so she can recover. However, plaintiff's recoverable damages are the amount of the judgment reduced by the percentage of fault attributed to the plaintiff. So Alice's recovery is $100,000-(50% of $100,000), which is $50,000.

Alice sues Bill alleging negligence. Bill defends, claiming that Alice's negligence also contributed to her injuries. At trial, the factfinder assigns 40% responsibility to Alice and 60% responsibility to Bill. The factfinder also finds that Alice's damages are $100,000. In a pure comparative fault jurisdiction, how much is Alice entitled to recover?

Alice is entitled to recover $60,000. Correct. Alice is entitled to recover only that portion of the judgment that is not attributable to her fault. So the judgment of $100,000 is reduced by 40%, the amount of fault attributed to Alice, which if $40,000. $100,000-$40,000=$60,000, making choice A correct.

Private citizen A publishes a letter to private citizen B about private citizen C. In the letter A tells B false statements about C, which hurt C's reputation. If C sues A for the harm caused to his reputation, which of the following is the best defense for A?

C A had a fiduciary relationship with C and C had a duty of confidentiality with regard to statements made by A. Correct. Publication requires communicating the defamatory statement to a third party. Communications within certain fiduciary relationships are not considered communications to a third party. Examples are client to lawyer, where confidentiality privilege applies, confessor to priest, patient to psychiatrist. If A's relationship with B was one of these then A would not have published the statement when the letter was sent to B.

Abe, who is negligently driving while intoxicated, is stopped at a red light. Betty negligently fails to stop and hits Abe's car in a rear-end collision. Abe sues Betty in negligence for personal injuries he suffered in the collision. In this comparative negligence jurisdiction, what effect will Abe's negligence have on his recovery?

C Abe's own negligence of being intoxicated does not affect Abe's recovery because Abe's intoxication was not a factual cause of his own injuries. Correct. Abe's own negligence will affect his recovery only if it was a factual cause of his injuries. This question asks about the requirement of causation as part of a comparative negligence defense. Restatement (Third) Torts § 4.

Petula lives in a residential neighborhood in State and keeps a Bengal tiger in her backyard as a pet. Rusty, Petula's good friend, decides to visit her Bengal tiger. Not realizing how dangerous the big cat can be, Rusty carelessly walks right up to the cage and puts his hand into the enclosure, taunting the animal. The animal becomes agitated, manages to escape from the enclosure, and mauls Rusty. Rusty then sues Petula in strict liability for the injuries he sustained as a result of being mauled by the tiger. Petula defends by claiming that Rusty was comparatively negligent. In a pure comparative fault jurisdiction, which of the following statements is correct?

B If Rusty has been comparatively negligent, his recovery in strict liability will be reduced according to his share of comparative responsibility. Correct. A plaintiff's recovery in strict liability may be reduced by the percentage of comparative fault assigned to the plaintiff. Restatement (Third) Torts § 25.

As part of a game of truth or dare Melissa walked toward a perfect stranger, and when she was about 10 feet from the stranger she asked the stranger the time. As the stranger looked up from his watch, Melissa removed an unloaded gun from her pocket, pointed it at the stranger, and pulled the trigger. As he saw this, the stranger yelled in terror, jumped back, and raised his hands. Melissa laughed uncontrollably at the stranger's reaction and then told him it was a simple prank and the gun was unloaded. Will Melissa be liable for assault and battery?

C Melissa will be liable for assault even though she was not attempting to commit a battery.

Ladder Co. manufactures kitchen stepladders for residential use. Dad used a Ladder Co. ladder to post a sign above the door of his home office, unaware that his five-year-old son was playing in the office. While Dad was standing on the ladder, his son suddenly opened the door, striking the ladder. Dad fell off the ladder, fracturing his hip. The ladder came with no warnings about not using it in front of an unlocked door. In a lawsuit brought by Dad against Ladder Co. for products liability, is Ladder Co. likely to be found liable?

B Ladder Co. is likely to be found not liable as a matter of law because a product seller is not subject to liability for failing to warn or instruct regarding risk that should be obvious to or generally known by foreseeable product users. Correct. A manufacturer of a product is not subject to liability for failing to warn about an obvious risk. This question asks whether a manufacturer of a product can be subject to liability for its failure to warn about a risk in using its product that is generally known.

B suddenly and violently attacks A with a machete. A evades the machete and kicks B in the stomach causing B to stumble down some stairs and to collide with C who unknown to A was coming up the stairs. The collision causes C to suffer a torn ligament in his knee. If A raises the self-defense would A be liable to C?

B A would not be liable to C because A acted reasonably in defending herself against B.

Alice sues Bill alleging negligence. Bill defends, claiming that Alice's negligence also contributed to her injuries. At trial, the factfinder assigns 50% responsibility to both Alice and Bill. The factfinder also finds that Alice's damages are $100,000. In this pure comparative fault jurisdiction, what amount, if any, is Alice entitled to recover?

B Alice is entitled to recover $50,000 from Bill because her recoverable damages are her damages reduced by the amount of fault attributable to her. Correct. Plaintiff's recoverable damages in a pure comparative fault jurisdiction are reduced based upon the percentage of fault attributed to the plaintiff.

Private Citizen is walking through the park and hears two "average people" having a conversation, in which one is telling the other that her brother has had an affair with a well-known federal Senator and that the Senator's spouse is unaware of the affair. Private Citizen posts "Senator (name) is having an affair and spouse is unaware," on his Facebook page and after a day has received in excess of 1 million likes. Senator sues Private Citizen for defamation. What is the evidentiary standard by which the Senator must prove actual malice in order to survive judgment as a matter of law?

B Clear and Convincing evidence Correct. In most civil cases the plaintiff need only prove their case by a preponderance of the evidence standard. However in a defamation case with a public official/public figure plaintiff actual malice must be proven by clear and convincing evidence. This higher standard is thought necessary to ensure the first vigorous and informed debate on public officials and figures. The first amendment is the Genesis of this higher standard.

While visiting the former Soviet Republic of Georgia an American Tourist tells a native Georgian in English that Tourist's traveling companion stole from the Georgian museum. The native Georgian does not understand a word of English and has no clue what Tourist said. However traveling companion, who speaks English heard what Tourist said to native Georgian. May traveling companion recover for defamation in a suit against Tourist if traveling companion did not steal anything?

B No, because Tourist did not publish the statement. Correct. Publication requires that the person to whom the statement was communicated understand the statement. Here the native Georgian did not understand anything Tourist said so there was no communication. Even though the plaintiff heard and understood what was said the communication must be to a person other than the plaintiff in order for publication to occur.

Jill loans her car for the evening to Friend who needs the car for social purposes. Jill knows that Friend's driver's license was suspended a month previously on account of repeated instances of reckless driving. In the course of the evening, Friend drives the car negligently and injures Pedestrian. In a cause of action against Jill brought by Pedestrian for recovery of her injuries, can Jill be held liable for negligence?

D Jill may be held liable to Pedestrian in negligence because the reasonable prudent person would likely not have lent her car to Friend, knowing that Friend's license was suspended for repeated instances of reckless driving. Correct. This question asks about negligent entrustment. See Restatement (Third) Torts § 19 and illustration 1 upon which this fact pattern is based.

Hotel Owner contacts Pipe Co. to deliver and install a large pipe to display a sign for its hotel. Hotel Owner thoroughly vets Pipe Co., and Pipe Co.'s representative sends Hotel Owner information showing that Pipe Co. has extensive experience in installing pipes like this and that Pipe Co. is bonded for this purpose. When Pipe Co.'s workers attempt the installation by using their truck, hooks, chains, and additional poles for stabilization, the pipe falls, bouncing on a passing car and injuring Driver. In a lawsuit brought by Driver against Hotel Owner for injuries sustained when the pipe fell on his car, may Hotel Owner be subject to direct or vicarious liability?

B Hotel Owner may not be subject to direct liability for the actions of Pipe Co. because Hotel Owner did not owe a duty of care with respect to the manner in which Pipe Co. performed its work and did not retain control over any part of its work. Correct. It is a correct statement regarding the direct liability of one who hires an independent contractor and does not retain control over any part of the contractor's work. Restatement (Third) Torts § 56.

Bouncer erroneously thinks that Patron is stealing money from a casino. Bouncer accosts Patron and escorts patron out the back door of the Casino. As bouncer opens the door to let Patron out of the building and into the alley he shouts, "Next time don't steal money you thief." A couple people in the alley overhear what Bouncer said. Assume damages are not an issue and Patron sues Bouncer for defamation. Will Patron prevail?

B Yes, if it was reasonably foreseeable that Bouncer's statement would be overheard Correct. If it was reasonably foreseeable that Bouncer's statement would be overheard then Bouncer would be deemed to have satisfied the negligence requirement of publication. Where a reasonable person would foresee a statement would be overheard and it is overheard then there is publication even if the publisher themselves may not have foreseen it would be overheard.

Coach of Professional Athlete thinks that Professional Athlete is taking illegal performance enhancing drugs after seeing athlete inject himself in the gym. Conflicted by what to do Coach retreats to his private office and decides to record on a digital dictation recorder what she observed. Coach speaks the following into a private recording device, "At approximately 2:45 pm athlete injected illegal performance enhancing drugs into her left thigh today in the gym. I am currently not sure if to confront athlete or to report athlete to regulatory board without telling athlete anything." Unknown to Coach a burglar was hiding in a closet in Coach's office and heard everything Coach said. The burglar later snuck out and told many people what Coach said. It turns out that Professional Athlete was injecting perfectly legal and prescribed antibiotics into her thigh. Which of the following is correct?

B Coach is not liable for defamation because Coach neither intended the burglar to hear the statement nor was it reasonably foreseeable that a burglar was hiding in the closet. Correct. We must initially decide if Coach published. If Coach did not publish then she cannot be liable for defamation. Here Coach did not publish because publication requires that the person issuing the statement must intend for another to receive the statement or the statement must be made such that it is reasonably foreseeable that the statement will be heard and understood by another person apart from the plaintiff. Here Coach thought she was alone in her private office making the statement and so Coach clearly did not intend for it to be transmitted to another person. It was likely also unforeseeable that a burglar was in the closet and therefore the transmission of the statement to another person was not foreseeable. This requirement of intent of transmission or foreseeability of transmission has often been called the intent and negligence requirement of publication. Here because there was no intent to transmit the statement and because it was not reasonably foreseeable a burglar in a closet would overhear it, there was no transmission. Hence there was no publication. There can be no defamation cause of action without publication.

Company hires Janitor to provide maintenance and janitorial services for its apartment building. When it hires Janitor, Company knows that Janitor has a record of inappropriate sexually aggressive conduct toward women. In his role as janitor, Janitor has frequent interactions with Tenant, one of the building's occupants. One evening, after his workday is over, Janitor knocks on Tenant's door. Being familiar with him because he is the janitor at apartment complex, Tenant lets him in, whereupon Janitor rapes her. Tenant sues Company in negligence, seeking to hold it liable for her injuries. Can Company be held liable for negligence?

B Company may be held liable in negligence in hiring Janitor for a job in which Janitor would have access to female tenants within their apartments. Correct. Janitor was a foreseeably dangerous employee, for which Company may be held directly liable.

A state administrative regulation prohibits railroad trains from blocking highway crossings for more than 10 minutes. Railway allows one of its trains to remain in a highway crossing for 30 minutes. Fifteen minutes into this 30-minute period, Driver, who is driving his car on the highway, fails to notice the train until it is too late and collides with it, suffering an injury. Driver sues Railway in negligence. At trial, Driver concedes that his negligence should reduce his recovery under comparative negligence principles, but claims that Railway is negligent per se for violating the regulation. The evidence at trial reveals that the history of the regulation and of the agency's findings accompanying the regulation show that the only purpose of the regulation is to encourage the free flow of traffic and prevent traffic delays. What effect, if any, does Railroad's violation of the regulation have on Railroad's liability for negligence?

B Driver should not be permitted to use the regulation's violation to establish Railway's negligence because the prevention of personal injuries is not part of the regulation's purpose. Correct. The facts explain that the regulation's only purpose was to encourage the free flow of traffic and prevent traffic delays. Therefore, it was not enacted to prevent the type of harm about which Driver is complaining, a car colliding with a train.

Driver is driving his car at 60 miles per hour on a stretch of highway with a 50 mile per hour speed limit when he runs into Pedestrian, seriously injuring her. In her negligence action against Driver to recover for her injuries, Pedestrian alleges that Driver was negligent in exceeding the speed limit. In order to establish factual causation (also known as causation-in-fact) as part of her negligence action, what will Pedestrian need to prove?

B Driver would not have hit her or would have caused her less harm if he had not been exceeding the speed limit at the time she was hit. Correct. This question asks about establishing factual causation (also known as factual causation or but for causation) in a negligence action. It is essential to a plaintiff's negligence action that she prove that the harm about which she is complaining would not have occurred absent the negligent conduct. Restatement (Third) Torts § 26.

Funny Business manufactures novelty items, including an exploding cigar made to blow up with a loud bang and the emission of smoke. Employee purchased the exploding cigar and presented it to Boss at an office birthday party. Boss lit the cigar. When it exploded, the heat from the explosion lit Boss's beard on fire, causing serious burns to his face. In a lawsuit brought by Boss against Funny Business, Boss alleges that he was injured by a defectively designed product. In a products liability action, may Funny Business be held liable?

B Even if the factfinder determines that there is no reasonable alternative design available that would provide similar prank characteristics, Funny Business can still be held liable in tort for the cigar having a defective design if the factfinder determines that the utility of the exploding cigar is so low and the risk of injury was so high that the cigar should not have been marketed at all. Correct. See Restatement (Third) Torts: Products Liability § 2(b).

Hospital negligently misidentifies a corpse, causing it to be cremated rather than sent to a funeral home for burial as directed by the family. As a result of Hospital's negligence, members of the family suffer serious emotional harm upon learning of the Hospital's mistake. In a lawsuit brought by the family against Hospital for its negligence in misidentifying their family member, may Hospital be subject to liability in negligence?

B Hospital may be subject to liability in negligence because the family suffered severe emotional injury due to negligent mishandling of a corpse. Correct. Typically, pure emotional distress alone is not compensable in a negligence action. However, there are exceptions, including when a defendant's negligence causes serious emotional harm during the course of an undertaking especially likely to cause serious emotional harm. Mishandling of a corpse is one of those undertakings.

Phone Co. lays underground cable throughout City. Phone Co. hires Indep as the independent contractor to perform the digging necessary for laying the cable. When the job starts, Indep has no previous digging or construction experience and only several hours of practice using a trenching machine. Prior to hiring Indep, Phone Co. did not inquire into Indep's experience using trenching machines or working around gas pipelines, the presence of which Phone Co. was reasonably certain to arise. During the course of the digging, Indep negligently strikes a gas line, causing an explosion that seriously injuries Tony. In a negligence action brought by Tony against Phone Co., will Phone Co. be subject to liability?

B Phone Co. may be subject to direct liability to Tony for failing to use reasonable care in selecting a competent contractor. Correct. Tony is seeking to hold Phone Co. directly liable for its negligence in choosing its independent contractor. This question asks about a principle's liability for the tortious conduct of its independent contractors. Although often a defendant is not subject to liability for the tortious conduct of its independent contractors, there are some exceptions to that rule. Restatement (Third) Torts § 55 and illustration 1 upon which this fact pattern is based.

Pool Co. manufactures aboveground swimming pools for residential use. Because the pools are only four feet deep, Pool Co. has posted warnings in large letters on the outside of their pools that warn "DO NOT DIVE INTO POOL. SEVERE HEAD INJURY RISK." While in a friend's backyard party, Mildred, age 25, reads the warning and chooses to dive head first into a Pool Co. pool. In doing so, she strikes her head on the bottom of the pool, suffering a severe head and spine injury. At trial in the tort action brought by Mildred against Pool Co. to recover for her injuries, experts testify that when Mildred's hands hit the pool's slippery vinyl bottom, her hands slid apart causing her head to strike the bottom of the pool very hard. Experts testify that the pool was designed to do so in order that people who may dive in would be guided away from the pool's bottom. The testimony is that this is the safest way to make an aboveground swimming pool. Experts also testify that the pool in which she was injured did not deviate from its intended design. Expert witnesses for both sides also agree that the vinyl pool liner used in the pool in which Mildred was injured was the best and safest liner available at the time and that no alternative, less slippery liner was feasible. At trial, the factfinder determines that the warnings were adequate. Should Pool Co. be held liable to Mildred?

B Pool Co. cannot be held liable to Mildred for her injuries because Mildred has failed to establish a defect necessary to impose liability on Pool Co. caused by its defective product. Correct. Mildred has failed to establish a defect necessary to impose liability on Pool Co. caused by its defective product. This question asks about the need for proving a defect in order to impose products liability upon a manufacturer. Restatement (Third) Torts: Products Liability § 2.

Private Citizen is falsely accused on social media of being a sympathizer to a fundamental religious terrorist group. The mainstream media also run the story and Private Citizen loses job as a result and is ostracized by his neighbors, peers and friends. The accusation it turns out was completely untrue and Private Citizen sues initial poster and media in defamation. Which of the following is true?

B Private Citizen cannot recover unless he proves that the statement was made with some level of fault. Correct. Public officials and public figure plaintiffs must prove that defamatory statements were made with actual malice to recover. However when the plaintiff is a private citizen suing to recover from a false statement of public concern (such as terrorist allegiance) then the plaintiff need only prove that the statement was published with some fault. Most courts construe this to mean that a private figure plaintiff suing in defamation based on a statement which is matter of public concern need only show that the publisher was negligent.

Soda Company, a company that provides beverages to local businesses, employs Delivery Person and has provided Delivery Person with detailed instructions on exactly how to conduct its business throughout the workday. Delivery Person is delivering beverages to SuperStore. Teenager is shopping at SuperStore when, while hauling sodas into SuperStore, Delivery Person carelessly runs over Teenager's foot, spraining and bruising it. In a negligence action brought against Soda Company on Teenager's behalf for the injuries sustained when Delivery Person ran over his foot, may Soda Company be held vicariously liable?

B Soda Company may be held vicariously liable for Delivery Person's negligence because Delivery Person was an employee at the time of the accident with Teenager. Correct. An employer may be held vicariously liable in negligence for its employee's tortious conduct.

Shopper insulted store clerk by saying that the store clerk had a disgusting body odor. Shopper intended to insult and embarrass store clerk and said it loud enough for the fellow shoppers in the vicinity to hear. Store clerk was mortified when other shoppers laughed. As a result of the insult, store clerk fainted and hit his head on a display and suffered a concussion. If store clerk brought an action for Intentional Infliction of Emotional Distress against shopper, would store clerk recover?

B Store clerk would likely not recover because the shoppers comment is legally insufficient to support a cause of action for IIED. Correct. Intentional infliction of emotional distress requires more than an act intended to cause severe emotional distress and the resulting distress. In order to be actionable, not only must the tortfeasor intend severe emotional harm but his actions which bring about the severe harm must also be "extreme and outrageous." The restatement has defined this standard as acts which are beyond all bounds tolerated by a civilized society. Mere insults have been held not to fall within the category of extreme and outrageous. Therefore even though the shopper intended to cause emotional harm and acted deliberately to achieve that purpose and did in fact achieve the intended result the insult is legally insufficient to sustain the action.

Shoplifter is seen placing an item in her coat pocket. As soon as shoplifter exits the store she is stopped by store security guards, who quietly and discreetly ask her to stop and empty her pockets. Did the guards act lawfully?

B The guards acted lawfully if their request to stop was reasonable. Correct. It states the rule of law known as the shopkeeper's privilege. A shopkeeper may reasonably detain a suspected shoplifter. Any unreasonable detention, whether in terms of use of force, duration or distance from the store is not privileged however.

In all jurisdictions that have replaced the traditional defense of contributory negligence with comparative fault:

B proof of the plaintiff's comparative fault may operate to reduce the plaintiff's recovery by the percentage of fault attributable to her. Correct. It is a correct definition of comparative fault.

Apple County maintained a nature park that included paths along its canyons and gorges. Park Ranger, employed by the county, became aware that a stream had dangerously eroded the support for a path, but neglected to close the path or to post warnings. A group of campers pitched camp after dark near the path. One of the campers, Charles, left the group to go on a walk. Charles fell into a gorge when the weakness in the support for the path gave way. The other members of the camping party heard what sounded like a falling object and called out to Charles, but received no response. Sean proceeded with a flashlight to investigate and discovered that Charles had fallen into the bottom of the gorge and appeared to be unconscious. In his attempt to descend to the bottom of the gorge, Sean lost his footing, fell, and suffered harm. Sean sues Apple County and Park Ranger in negligence for his injuries. Who, if anyone, may be liable to Sean for his injuries?

Both Apple County and Park Ranger may be liable to Sean for his injuries because his rescue was a foreseeable consequence of their negligence. Correct. When a person's tortious conduct imperils another the actor's liability generally includes injury suffered by the rescuer in effectuating a reasonable rescue. Restatement (Third) Torts § 32. This question asks about the ability of rescuers to recover from an original tortfeasor. In the famous words of Judge Cardozo, "danger invites rescue."

Jessica was driving her car, which was manufactured by Motor Company, when the voltage regulator in the car failed due to its negligent installation. The failure caused the battery fluid to boil, which produced toxic fumes that reached the interior of the car. Jessica suffered chronic vocal-cord damage as a result. Prior to this occurrence, Jessica was a popular singer who earned several millions of dollars each year. Jessica sues Motor Company for its negligence to recover for her injuries. Will Jessica be able to recover for her injuries?

C All of Jessica's lost earnings due to her vocal-cord injury are within the scope of Motor Company's liability because the type of injuries Jessica suffered were foreseeable to Motor Company from its negligence. Correct. Jessica's lost earnings due to her vocal-cord injury are within the scope of Motor Company's liability were foreseeable to Motor Company from its negligence. This is an application of the eggshell skull plaintiff rule.

Blasting Co. is in the business of razing buildings for districts that are building up their communities. Blasting Co. is aware that there are dangers inherent in using dynamite to raze buildings, and although it adopts all reasonable precautions, it is likely that there will be some unintended damages caused by way of flying debris or the concussion of the blast. Blasting Co. has been employed by City to raze several buildings near downtown. Several miles away, Kylie has been farming minks for their fur. Unfortunately, mink are very sensitive and have very nervous dispositions, and after feeling the vibration of the blasting from many miles away (which vibrations were not detectable to humans), several of Kylie's female minks that had just given birth ate their offspring. Kylie sues Blasting Co. in strict liability for the loss of the baby minks. Should Blasting Co. be subject to strict liability?

C Blasting Co. should not subject to strict liability for the injuries to Kylie's mink farm because the destruction of mink kittens by their mothers is not the type of damage that makes blasting an abnormally dangerous activity. Correct. The destruction of mink kittens by their mothers is not the type of damage that makes blasting an abnormally dangerous activity.

Client was being represented by Attorney in a civil lawsuit. As part of the representation, Client told Attorney that her significant other was addicted to painkillers. Attorney later told a third person that Client's significant other was addicted to painkillers. It turned out that Client's significant other was not addicted to painkillers. If Client is sued by her significant other for defamation which of the following is a valid defense for the Client?

C Client is not liable because there was no publication. Correct. When a communication is made to a person who is bound by confidentiality such as a lawyer, psychiatrist or priest in confession then the person making the statement is not deemed to have published. This is because no reasonable person would have expected the priest, lawyer or psychiatrist to repeat the statement. Because telling one of these entities who is legally bound to confidentiality is like telling no one there is no intent to publish and therefore no publication.

Nala keeps a young lion in her backyard in order to scare off any intruders. The lion is carefully fastened to a post by a chain. Because of a defect in the chain that Nala could not be expected to detect, the lion breaks free and mauls Neighbor on the sidewalk, injuring him. In a lawsuit brought by Neighbor against Nala, may Nala be subject to liability?

C Despite her exercise of reasonable care in attempting to confine the lion, Nala may be held strictly liable to Neighbor for his injuries. Correct. A possessor of a wild animal may be liable for injuries caused by the animal without regard to due care used.

Doctor, a clinical psychologist, becomes aware during the course of counseling, that his patient Dexter is sexually abusing Dexter's eight-year-old stepdaughter, Kylie. Doctor does not communicate this information to Kylie's mother or appropriate officials. He also does not take any other steps to prevent Dexter from continuing his sexual assaults of Kylie. When Kylie's mother discovers the abuse, she sues Doctor on behalf of Kylie for his negligence in not telling her or others. Which of the following statements regarding Doctor's liability in negligence for failure to warn is correct?

C Doctor owes a duty of reasonable care to Kylie and may be subject to liability for the harm due to Dexter's continuing abuse of her. Correct. A mental health professional and his patient have a special relationship, which may require the health professional to act affirmatively. Restatement (Third) Torts § 41.

Patient gives consent to be treated by surgeon for a minor surgery on plaintiff's left elbow. While plaintiff is unconscious from the anesthetic and during the course of the surgery on the left elbow the surgeon decides to examine the patient's entire body. Surgeon finds nothing else wrong with patient. Patient sues the doctor in battery. What is the best defense of the doctor?

C During the surgery, the doctor realizes the patient suffered from a condition that typically rapidly spreads to all parts of the body and only a careful and painful examination of the patient's entire body would reveal whether or not other parts of the body were affected needing immediate treatment. Correct. If during surgery the doctor realizes that the patient suffers from a condition that rapidly spreads to other parts of the body and if the patient needed immediate treatment to prevent spread of the disease or the doctor discovers the need for immediate treatment during surgery then the doctor would be acting reasonably if she treated the patient's condition. The doctor would not be committing a battery if it was reasonable to assume consent for treatment would have been given if the patient were conscious and had the opportunity to consent.

Jeremy purchased a bottle of liquor from Liquor Barn. The liquor was bottled by Bottlers Inc., in bottles manufactured by A#1 Glass Co. While Jeremy was opening the bottle, it suddenly exploded, causing disfiguring cuts to his face. The fact finder determines that, originating with A#1 Glass Co., the bottle contained a weakness in the glass structure that caused Jeremy's harm and was a departure from the product's intended design. The weakness in the bottle's structure caused the bottle to explode. In a lawsuit brought by Jeremy against Liquor Barn, A#1 Glass Co., and Bottlers Inc., which of the defendants may be subject to liability for the defective product?

C Liquor Barn, Bottlers Inc., and A#1 Glass Co. may all be subject to liability even though they exercised reasonable care in the preparation and distribution of the defective bottle of liquor. Correct. The general rule is that all those engaged in the business of selling or distributing a defective product may be subject to liabilities for injuries to persons or property as a result. Restatement (Third) Torts: Products Liability § 1.

Mom is the mother of Toddler, who is almost two years old. Mom and Toddler are visiting Friend's quaint rustic cabin. One morning when Friend has gone to run errands, Mom and Toddler are in the kitchen, a lovely room lit by a kerosene lamp sitting on a kitchen table. Toddler knocks the lantern over during a less than 10-second period during which Mom has turned her back in order to take a boiling pot off the stove. The knocked over lantern starts a fire that damages Friend's cabin. In a negligence action brought by Friend against Mom, is Mom likely to be held liable in negligence?

C Mom is likely not liable in negligence if turning her back for that brief period of time was reasonable. Correct. Mom will not be liable in negligence absent a showing that her conduct was unreasonable and her unreasonable conduct caused the fire. Because she took her eyes off her child for such a short period of time and did so to eliminate a dangerous situation, Mom's conduct may be reasonable under the circumstances, making C correct. B is an attractive option, but C is the better answer.

Pedestrian was walking along the street on the edge of Homeowner's property. Unknown to Homeowner the door to his house was open and he was unaware that Pedestrian was on the street. In an instant Homeowner's massive and ferocious dog sped through the door and charged towards Pedestrian. As Pedestrian looked across the 15 yards of lawn between the street and the doorway he was petrified, and screamed in horror as the dog bound towards him. The dog sprinted towards Pedestrian baring it's teeth and barking ferociously. Pedestrian froze with fear but just before as the dog reached the edge of the property less than 2 feet from where Pedestrian was in the street it suddenly stopped and moved back away from the property line and sat on the lawn. Relieved, Pedestrian walked away scared and almost in shock because the fear he had just experienced. Homeowner knew the dog would not reach Pedestrian because of the buried electronic fence installed in the yard which was connected to the dog's collar and prevented the dog from reaching the street. Did homeowner assault Pedestrian?

C No, because Homeowner did not act with intent

During a debate on gun rights on the floor of the federal House of Representatives, Representative 1 who is in favor of reducing access to guns, knowingly makes a false statement about Representative 2, who is a gun rights advocate. Representative 1 said, "Why don't you tell us the real reason you support gun rights. Let everyone know that you are a majority shareholder in the largest gun manufacturing corporation in the country." The truth is that Representative 2 has no shares in the gun manufacturing corporation and has no other agenda other than applying faithfully what she thinks are the mandates of the Second Amendment of the Constitution. A newspaper reporter attended the legislative debate. He takes notes. A few days later the newspaper where reporter is employed runs a story on the statement made by Representative 1. As part of the story the newspaper quotes Representative 1. May Representative 2 sue the newspaper if the newspaper knew of the falsity of Representative 1's statement at the time it published the story?

C No, because this was a fair and accurate report of a public proceeding. Correct. While it is true that re-publishers are liable for falsities that they republish, the press enjoys the privilege of fair and accurate reporting. This privilege is an absolute privilege and is based on the idea that the press is the agent of the people in a democratic system. The privilege may be defined as follows, the press is immune from liability for defamatory statements they publish when the publication is a fair and accurate report on public proceedings and meetings.

A state statute requires that all slow-moving vehicles drive as far to the right on the highway as possible. The statute's primary purpose is to minimize the safety hazards posed by vehicles whose slow speed can interfere with the traffic flow. Driver is driving slowly on a highway heading north and is violating the statute by driving in the fast lane on the left. Passenger is riding along in her car. Unexpectedly, a car heading south on the highway crosses the median line striking Driver's car and injuring Passenger. Had Driver been in the lane on the far right, her car would have avoided contact with the other car. Passenger sues Driver, seeking to establish her negligence by proving her violation of the statute. What effect, if any, may proof of Driver's violation of the statute have in a negligence action brought by Passenger?

C Passenger will likely not be able to use the statute to prove Driver's negligence because the statute was designed to prevent accidents between cars moving in the same direction, not to prevent accidents between cars moving in opposite directions. Correct. The statute was not designed to prevent the type of accident that occurred in this case and therefore cannot be used to establish the standard of care. Restatement (Third) Torts § 14.

During a debate on gun rights on the floor of the federal House of Representatives, Representative 1 who is in favor of reducing access to guns, knowingly makes a false statement about Representative 2, who is a gun rights advocate. Representative 1 said, "Why don't you tell us the real reason you support gun rights. Let everyone know that you are a majority shareholder in the largest gun manufacturing corporation in the country." The truth is that Representative 2 has no shares in the gun manufacturing corporation and has no other agenda other than applying faithfully what she thinks are the mandates of the Second Amendment of the Constitution.Which of the following is the reason Representative 1 may not be sued successfully for defamation by Representative 2?

C Representative 1's statement was made as part of a legislative body's proceeding. Correct. Any statement made as part of a legislative body's proceeding is absolutely immune from a defamation cause of action. Unlike the judicial privilege the statement does not have to be related to the proceedings. The legislative privilege in that sense is broader than the judicial privilege.

On the floor of the United States Senate during a debate on gun rights a senator stated she didn't think guns should be available without a background check because even people like the United States Secretary of Defense ("Secretary") got angry when hunting and shot their friends deliberately. The truth is that the Secretary accidentally shot a friend while he was hunting. In response, at a press conference in his office, the secretary of defense told the press the following day, "We feel compelled to respond to the Senator's comment, in part to clear my name and in doing so we would like to mention we know that she takes bribes from the big contractors who hate guns?" The secretary's statement at the press conference was false. If the senator sues the secretary for defamation which of the following is correct?

C The Secretary's statement may be privileged. Correct. It recognizes the existence of other privileges besides the legislative privilege, such as the public official privilege or even the poorly defined newsworthiness privilege, which may be operating here to protect the statement, even though it was not made on the floor of the legislature.

BPV Co. manufactures bulletproof vests for use by police and security personnel. BPV Co. offers several different models, some providing front and back protection only, and others providing wrap-around protection. State highway patrol officials chose to purchase the model that provides front and back protection only. They did so because that model is less expensive, allows greater flexibility of movement, and is more comfortable. State Trooper was shot and killed while making a routine traffic stop. The bullet entered the side of his body, where the vest did not provide protection. In a lawsuit brought on behalf of State Trooper's estate against BPV for injuries caused by an allegedly defective product, the estate's lawyer argues that the design of the vest is defective because it does not provide wrap around coverage. In the products liability lawsuit against BPV for State Trooper's death, what is the legal relevance of the alternative of the wrap around vest design?

C The differences in advantages and disadvantages of the wrap-around vest and the front-and-back only vest are sufficiently understood by consumers does not render the front-and-back design in a condition unreasonably safe. Correct. Whether a product has a defect in design turns on many factors, including for example the existence of an alternative design, the increase in cost to incorporate the alternative design, and the usefulness of the product if it incorporates the alternative design. The differences in advantages and disadvantages of these two products would be understood by consumers and also become part of the defective design calculus.

Two students at the end of the semester got a D grade in torts. One student was incredibly irate about the grade and the other was blissfully happy he did not get an F. The blissfully happy student decided to celebrate. In order to celebrate she went to the store and bought a bow and arrow. She waited until the classroom was full and began shooting suction cup tipped, wooden arrows into the air in the classroom with a sign on them that said, "Joy, Joy I will work harder because now I know that your grades in law school are proportional to the amount of work you do." Her only goal in shooting the arrows was to share her profound discovery about the correlation between grades and work in law school. However, the arrows bounced off the walls and hit many, many people causing them to experience offensive contact. The irate student got the same kind of bow and arrow that the other student got and began shooting the students who did better than he did. Using his 10 years of archery lessons, he methodically aimed the suction cupped arrows at the individual students and let fly. The students he aimed at experienced offensive contact from the arrows. Which of the following about the students' intent is true?

C The irate student acted with intent because he acted with the specific purpose of causing harmful and offensive contact.

Farmer is a country farmer who has a lower IQ than most individuals. However, he is also unusually physically fit, much physically bigger and stronger than the average person. Lately, as part of his trade as a farmer, he has been stacking hay on his property. However, he has been doing so in a way that a farmer of ordinary intelligence would know will cause the hay to combust spontaneously into flames. Defendant's hay does in fact combust causing a fire, which spreads to Neighbor's property causing property damage. In the negligence action brought by Neighbor against Farmer, how should the court instruct the jury regarding the reasonable prudent person standard of care?

C The reasonable prudent person standard will not take into account that Farmer has a lower IQ than most individuals. Correct. The reasonable prudent person standard does not take into account mental incapacities of the defendant.

In the middle of a trial for breach of contract the defendant businessman is called as a witness. During the course of the trial, the Defense lawyer asks businessman if he had ever done business with the Plaintiff. In response to the question businessman responded with a rant about how his housekeeper had been stealing from him for many years and tried to kill his dog at least three times. Assume also that the statement about the housekeeper is untrue and that the housekeeper's performance is legally irrelevant to the trial. Which of the following is the best argument for why the judicial privilege should not apply to the statement in a subsequent defamation lawsuit by housekeeper against business man?

C The statement about the housekeeper is unrelated to the trial. Correct. In order for the judicial privilege to apply the statement has to have been made in a judicial proceeding and be related to the proceeding. Answer choice C clearly states the statement is unrelated to the trial and therefore it forecloses application of the judicial privilege.

Toddler, a two-year-old child suffering from a bacterial infection, received a routine childhood vaccination with a vaccine manufactured by Vaco. Shortly after receiving the vaccine, Toddler spiked a very high fever, went into respiratory arrest, and died. Toddler's estate sues Vaco in negligence. At trial, Toddler's estate provides competent evidence that Toddler died due to a combination of the infection and the vaccine, and if either toddler had not had an infection or if Toddler had not been vaccinated, death would not have ensued. In its defense, Vaco presents evidence that the infection and its drug do not react together, but rather are entirely independent. Therefore, Toddler's death could only have been caused by the infection or the drug, but not a combination of both. The factfinder is persuaded by the defense's evidence and finds that the infection more likely than not caused the death of Toddler rather than the vaccine. Which of the following statements regarding the factual causation element of the negligence action is correct?

C Toddler's estate has failed to satisfy its burden of persuasion that the vaccine was a factual cause of Toddler's death. Correct. The factfinder concluded that the infection more likely than not caused the death of Toddler rather than the vaccine. Toddler's estate put on evidence from which the factfinder could have concluded that the death was caused by the vaccine, so Toddler's estate satisfied its burden of production. This question is more about the burden of persuasion rather than the burden of production. Had Toddler's estate satisfied its burden of persuasion, the factfinder would have found that the vaccine more likely than not caused the death of Toddler, but the factfinder did not.

Visitor was a customer of Barge, when while on the deck of Barge he accidentally fell into the water as the barge approached the pier. Employee of the barge hears Visitor's cries as he yells from the water that he cannot swim. Employee threw a heavy line (used to tie the barge to the pier) in the direction of the cries. The line came within two feet of the spot of Visitor, but Visitor did not grab the lines. Visitor was carried away from the pier by the outgoing tide and downed. At the time of this incident, the employee was aware that several life preservers were on a rack located near the front of the barge where Visitor fell overboard. The preservers remained in the rack and were not used. If the preservers had been used, Victim probably would not have drowned. Visitor's estate sues Barge in negligence, alleging that the failure to use the life preservers was the factual cause of Victim's drowning. Is Visitor's estate correct that Barge's failure to use the life preservers was a factual cause of Victim's drowning?

C Yes because not throwing the preservers to Victim was a substantial factor in bringing about Victim drowning. Correct. Negligence (breach of the standard of care) is a factual cause of a plaintiff's injury if it is more likely than not that the harm would have been averted but for the negligence of the defendant.

Curtis was a guest at Hotel where at 9:00 p.m. one evening there was a power outage that deprived Hotel of electricity. As there was no emergency lighting in his dark room, Curtis fell and injured himself. Curtis sued Hotel for his injuries, alleging that Hotel was negligent in not having inexpensive battery-powered lighting fixtures installed in its guest rooms. Curtis alleges that if his room had such a fixture, he would have avoided injury. The only expert to testify at trial explains that at the time of Curtis's injury, it was not customary in the hotel industry for in-room emergency lighting of this sort to be provided. May Hotel be subject to liability in negligence although if it was in compliance with customary hotel industry standards by not providing in-room emergency lighting?

C Yes, Hotel may be liable for Curtis's injuries despite the industry custom if requiring emergency lighting in hotel rooms is reasonable with respect to the risk of guests injuring themselves during a power outage. Correct. This question asks about the use of custom as evidence of a defendant's negligence. A defendant's compliance with customary standards does not preclude a finding of negligence. Restatement (Third) Torts § 13.

While asleep one night, homeowner hears a noise coming from a window in the house. She immediately goes down stairs armed with a handgun. In the dark she notices a broken window and a person climbing through the window. Without asking questions she aims at the intruder and fires, severely injuring the intruder in the process. It turns out that the intruder was homeowner's cousin who was intoxicated was simply breaking into the house to leave a present for the family. Will homeowner be able to claim the privilege of self-defense in a suit by cousin against homeowner?

C Yes, because the castle doctrine allows a person to use deadly force to protect their occupied dwelling at night.

Pat whispers a defamatory statement about Tracie to Francis. Assuming all three are private citizens and the defamatory statement is not about a matter of public concern, is the statement actionable?

C Yes, if Francis can prove pecuniary or special damages. Correct. In cases of slander (spoken defamatory statements) the plaintiff must prove special or pecuniary damages in addition to the other elements of defamation.

Billy has a pet goat, which he keeps on his property in a carefully constructed enclosure. Billy uses all due care to make sure that his goat does not escape, but one evening, Billy's goat does get loose and wanders onto Joan's property. It is dark as the goat walks normally around Joan's property. Joan is also out taking a stroll. She does not see the goat, stumbles over it, and falls, suffering an injury. In a lawsuit brought by Joan against Billy for her injuries when she tripped over Billy's goat, may Billy be subject to liability?

D Billy may not be subject to strict liability for Joan's injuries because the harm that Joan suffered was not part of the risk underlying the rule for strict liability for non-domesticated animals. Correct. A possessor of livestock that intrudes on another's land may be subject to strict liability if the animal escapes and causes harms that are an appreciable part of the risk justifying imposition of strict liability. Restatement (Third) Torts § 21.

Company provides a company-owned car to Harry for personal and business use. While Harry is on vacation, Harry is speeding while driving the car. He loses control, destroying a roadside billboard owned by Announcements, Inc. Announcements sues Company, claiming that Company negligently entrusted the car to Harry. Announcements claims that despite knowing of Harry's history of speeding, other moving violations, and a number of accidents, Company provided Harry with the car. Should Company be held liable in negligence for the destruction of Announcement's billboard?

D Company should be held liable in negligence because one of the risks that made Company negligent was that Harry would drive poorly and cause an accident. Correct. The risk that made Company negligent was that Harry would drive poorly and cause an accident, thereby giving rise to Company's direct liability.

Private citizen A publishes a letter to private citizen B about private citizen C. In the letter A tells B false statements about C which hurt C's reputation. If C sues A for the harm caused to his reputation may C recover?

C may recover for libel. Correct. A defamation law suit is actionable when a defendant publishes to a third party a false statement containing defamatory material of and concerning the plaintiff. Defamatory material is anything that harms the plaintiff's reputation. Defamation causes of action are actionable even without physical harm. When, as in this example, the defamation action is based on a written statement the appropriate cause of action is libel.

As Plaintiff is walking down the street, a chair falls out of Warehouse's window, striking Plaintiff on the head. Plaintiff seeks to sue Warehouse in negligence and wants to employ the doctrine of res ipsa loquitur to do so. When using the doctrine of res ipsa loquitur to prove a Warehouse's negligence, Plaintiff:

C must prove that the accident does not ordinarily occur in the absence of negligence. Correct. Res ipsa loquitur requires proof that the accident is the type of accident that does not ordinarily occur in the absence of negligence. Restatement (Third) Torts § 17.

Tracie and Francis were soccer rivals. One day before a game between their respective teams Tracie whispered to Francis, "I am going to beat you down immediately after this 90 minute game is over if you score. Two minutes into the game Francis scored a goal. Immediately after scoring he experienced nervousness and became fearful of what Tracie would do to him after the game. Surprisingly, Francis fainted as a result of the fear, hit his head on the pitch, and had to be carried off unconscious in a stretcher. Should Francis sue for assault and/or battery?

D Francis should not sue for assault nor should he sue for battery.

Alice sues Bill alleging negligence. Bill defends, claiming that Alice's negligence also contributed to her injuries. At trial, the factfinder assigns 50% responsibility to both Alice and Bill. The factfinder also finds that Alice's damages are $100,000. In a modified comparative fault (50% or "not as great as") jurisdiction, what amount, if any, is Alice entitle to recover from Bill?

D Alice is completely barred from recovering from Bill because she was 50% at fault. Correct. Plaintiff's fault is as great as that of the defendant, so she is barred from recovering, making D the correct answer because in a modified fault (50% or "not as great as") jurisdiction, when Plaintiff's fault is as great as that of the defendant, she is barred from recovering.

A city ordinance requires that all apartments be equipped with fire sprinklers in each sleeping area and in each stairwell leading to an occupied area. Apartment Owner, who owns an apartment building and leases apartments, hires Ivan, an independent electrical and plumbing contractor, to install fire sprinklers throughout the building. Ivan does not install a fire sprinkler in every stairwell as specified by the ordinance. Tenant, a tenant in Apartment Owner's building, is injured in a fire that started in a stairwell without sprinklers. Tenant would have escaped the fire without injury if a fire sprinkler had been in place. In a lawsuit brought by Tenant against Apartment Owner for injuries sustained in the fire, may Apartment Owner be subject to vicarious liability?

D Apartment Owner is subject to vicarious liability for Tenant's injuries because Apartment Owner hired Ivan for an activity for which a city ordinance imposed an obligation on Apartment Owner and Ivan's failure to comply with the city ordinance was a factual cause of Tenant's injury. Correct. It is a correct statement of vicarious liability. This question asks about the liability if a defendant who hires an independent contractor to perform an activity that is imposed on the defendant by statute or regulation. See Restatement (Third) Torts § 63 and illustration 1 upon which this fact pattern is based.

As Pedestrian was walking along the street on the edge of Homeowner's property, Homeowner decided to play a prank on Pedestrian. Homeowner opened the door of his house facing the street where Pedestrian was walking. He stood in the open doorway with his huge ferocious dog and called to Pedestrian, "Hey." As Pedestrian looked across the 15 yards of lawn between the street and the doorway he was petrified, and yelled to Homeowner, "make sure you hold on to that monster." At which point Homeowner yelled ATTACK!!!!!!!! and released the dog The dog sprinted towards Pedestrian baring it's teeth and barking ferociously. Pedestrian froze with fear but just before as the dog reached the edge of the property less than 2 feet from where Pedestrian was in the street it suddenly stopped and moved back away from the property line and sat on the lawn. Relieved Pedestrian walked away scared and almost in shock because the fear he had just experienced. Homeowner knew the dog would not reach Pedestrian because of the buried electronic fence installed in the yard which was connected to the dog's collar and prevented the dog from reaching the street. If Pedestrian sues Homeowner for battery Pedestrian will:

D Lose because Homeowner did not cause any harmful or offensive contact to Pedestrian

Max owns a furniture store and employs Alex to deliver furniture to retail customers. Alex's duties include entering customers' homes to situate items they have purchased. Having entered Customer's home to deliver a sofa, Alex assaults Customer. Prior to employing Alex, Max conducted no check of Alex's background, although a reasonable employer of an employee who would be entering the homes of customers would have conducted such a check. Had Max done so, Max would have discovered criminal convictions for assault and would not have employed Alex to make deliveries. Customer seeks to hold Max directly liable in negligence for Customer's injuries. Can Max be held directly liable for Customer's injuries?

D Max may be held directly liable for Customer's injuries because the injuries for which Customer seeks recovery are foreseeable from Max's negligence in not conducting a background check. Correct. The employer may be subject to direct liability under these circumstances for his negligence. This question asks about holding an employer liable for its negligence in hiring an employee. An employer may be held directly liable for its employee's wrongful conduct when the employer was negligent in selecting the employee. Restatement (Third) Agency § 7.03.

Mom is the mother of Toddler, who is almost two years old. Mom and Toddler are visiting Friend's quaint rustic cabin. One morning when Friend has gone to run errands, Mom and Toddler are in the kitchen, a lovely room lit by a kerosene lamp sitting on a kitchen table. When Mom leaves the kitchen for an hour to read a book, Toddler knocks over the lantern, starting a fire that damages Friend's cabin. In a negligence action brought by Friend against Mom, is Mom likely to be held liable in negligence?

D Mom is likely to be held liable in negligence to Friend because Mom failed to act as the reasonable prudent person when she left Toddler alone for an hour next to a kerosene lamp. Correct. It is unreasonable to leave a small child alone next to a kerosene lamp for an hour merely to go read a book, and the resultant fire was reasonably foreseeable from Mom having done so.

Lilly was a guest at Motel, which was located in a neighborhood where significant violent crime existed. After Lilly returned to her hotel room, Don was able to gain entrance to Lilly's room because the lock on the door was of the simple residential type that could be easily defeated with a credit card. After gaining entrance to Lilly's room, Don sexually assaulted her. Lilly sues Motel, claiming that it was negligent in providing inadequate locks for its guest rooms. May Hotel be held liable in negligence?

D Motel may be held liable in negligence because Don's criminal act was foreseeable from its negligence. Correct. The initial tortfeasor may be held liable when the criminal act was foreseeable as it was here. This question asks about intervening causes and proximate causation determinations. When an intervening cause is unforeseeable, unusual, or highly culpable, the initial tortfeasor is no longer a proximate cause of the plaintiff's harm.

About two years ago, Client was a patient of Ophthalmologist. Client alleges that during the time that he was Ophthalmologist's patient, Ophthalmologist improperly performed LASIK surgery on Client's eyes. The LASIK surgery was supposed to correct Client's poor vision. Instead, the surgery was performed skillfully, but resulted in blindness in both eyes, a risk about which Client was never informed and a reasonable person would not have known. If Client had known of the risk of blindness, he would not have opted for the surgery. Client hires Attorney Matthew to sue Ophthalmologist for medical malpractice. Regarding the underlying medical malpractice action, which of the following statements concerning the Ophthalmologist's liability is correct?

D Ophthalmologist's failure to inform Client of the material risks and available alternatives to LASIK treatment is a breach of the standard of care in negligence. Correct. This question asks about the informed consent doctrine in negligence law. A physician can be subject to liability in negligence if he fails to inform his patient of the material risks and alternative treatments to the patient's treatment protocol. Failure to inform constitutes a breach of the standard of care in negligence.

Sam received a text message with a video from an undercover agent pretending to be a gang member. Video showed a distant relative of Sam being severely beaten by other gang members a few days earlier. Distant relative has a viable cause of action for assault and battery against gang members. Upon seeing the video Sam got sick, vomited quite a few times, and was diagnosed with high blood pressure as a result of the video. He also experienced severely disabling emotional distress as a result of the video. May Sam maintain an action for intentional infliction of emotional distress against the gang members based on his reaction to seeing the video?

D Probably not, because Sam was not sensorily and contemporaneously present during the beating of his distant relative. Correct. One of the nuances of bystander recovery pursuant to IIED based on tortious conduct directed at another person is that the bystander must have observed the conduct as it is happening. According to the Dobbs Hornbook § 307 the line is drawn, "against recovery by excluding those who are not present and who have not had an immediate sensory perception of the injury." Here, Sam was not present and he received the video "a few days" after the "primary injury." Therefore, he would not be able to maintain the IIED action based on the tortious conduct directed to his distant relative.

After a legislative debate on gun rights on the floor of the federal House of Representatives, Representative 1 who is in favor of reducing access to guns, knowingly makes a false statement about Representative 2, who is a gun rights advocate at a press conference. At the press conference Representative 1 asks, "Why don't you tell us the real reason you support gun rights. Let everyone know that you are a majority shareholder in the largest gun manufacturing corporation in the country." The truth is that Representative 2 has no shares in the gun manufacturing corporation and has no other agenda other than applying faithfully what she thinks are the mandates of the Second Amendment of the Constitution.Is Representative 1 immune from a defamation lawsuit?

D Representative 1's is not immune from a defamation lawsuit because his statement is not covered by the legislative privilege. Correct. In order to be protected by the legislative privilege a statement must have been made as part of a legislative body's proceedings. Here the statement was made during a recess in proceedings and so the privilege would not apply.

Defendant, driving a car, approaches Plaintiff's car, and properly steps on the brakes. Defendant's brakes fail to function because she carelessly maintained the brakes. Defendant's car continues forward. In this emergency, Defendant chooses to step on the brakes again, and they fail again, causing Defendant's car to strike Plaintiff's car, injuring Plaintiff. Will Defendant likely be held liable for negligence?

Defendant will likely be held liable in negligence because her brakes failed because she unreasonably failed to properly maintain her brakes. Correct. This question asks about a person who finds herself in an emergency situation, which was created by her own negligence. Defendant has negligently failed to maintain her brakes.

Electric Company has strung power lines along the bank of a local river approximately 20 feet above the water's level. Recreational boaters, with masts sometimes exceeding the power lines by no more than one foot, often sail up and down the river. Polly is a guest on the boat of Friend, who is unfamiliar with this particular segment of the river. The height of the mast of Friend's sailboat reaches 21 feet above the water line. As the boat approaches the shore, Polly (who does not see the power line) is holding the mast when it comes into contact with the power line above. Due to the contact, Polly suffers severe electrical burns. In Polly's negligence action against Electric Company, the evidence shows that the likelihood of contact between sailboat masts and the power lines in this segment of the river is considerable, the severity of injuries when such contacts occur will probably be extreme, and the cost of raising the height of the power lines another 3-4 feet higher would be moderate. Can Electric Company be held liable in negligence?

Electric Company can be held liable in negligence to Polly for failing to raise the height of its power lines because Electric Company's cost of raising the power lines 3-4 feet higher is moderate as weighed against the considerable likelihood of contact between a mast and the lines and the severity of probable potential injuries. Correct. The facts tell us that the cost of guarding against the harm was moderate as compared against the considerable likelihood of a severe injury. This question asks whether Electric Company failed to exercise reasonable care. Restatement (Third) Torts § 3 explains that the primary factors to consider in determining whether a party was negligent are the foreseeable likelihood that the person's conduct will result in harm, the foreseeable severity of any harm that may ensue, and the burden of precautions to eliminate or reduce the risk of harm.

Parker parks his car at the top of a driveway, which is on an incline. Two minutes later, the car rolls down the incline and injures Pedestrian who was on the sidewalk. In suing Parker, Pedestrian seeks to rely on res ipsa loquitur in order to prove Parker's negligence. Which of the following statements about res ipsa loquitur is correct?

If permitted to employ the doctrine of res ipsa loquitur, the jury may infer that Parker was negligent. Correct. This question asks about the use of res ipsa loquitur to establish a defendant's negligence. If allowed to use the doctrine of res ipsa loquitur, the jury is permitted but is not required to infer that the defendant was negligent. See Restatement (Third) Torts § 17 and illustration 2, upon which this question was based.

Homeowner and neighbor own unfenced, adjoining pieces of property. Homeowner accidentally begins parking his car on neighbor's property because he is confused about the boundary line. After a few days of this, neighbor drags homeowner's car back onto homeowner's property and unintentionally ruins the transmission as he does so. Homeowner sues neighbor for trespass to chattel. What result?

Neighbor will prevail if he used reasonable force in moving the car.

For almost 2 centuries Landowner's family has owned a 2 miles stretch of coastal land just landward of a public beach. As the land has been passed down from generation to generation landowner's family has always allowed the public to walk on the land to get beach access even though there are many conspicuous signs saying private property on the land. The newest descendant to own the land has decided that he no longer wants anyone to access the land without permission. One day after this, beachgoer, as other beachgoers have done for centuries walks on the land to get to the beach. Did beachgoer trespass?

No, because a custom existed which permitted entry onto the land that would otherwise be trespass. Correct. If a custom exists which permits entry which otherwise would be a trespass then before entry is actionable the landowner must affirmatively provide notice that the consent is withdrawn.

Late one night, Pete places his loaded pistol on the coffee table in the family room of his home that he shares with his wife and two young children. The next afternoon Pete's 10-year-old son, Nolan, is spending time in the family room with his friend, Buddy. Nolan picks up the gun and plays with it, accidentally causing it to discharge. Buddy is injured by the gunshot. In a negligence action against Pete brought on behalf of Buddy, can Pete be held liable for negligence?

Pete may be held liable in negligence because Pete carelessly provided Nolan access to the gun in circumstances in which it was foreseeable that Nolan might use it improperly. Correct. When Pete carelessly leaves his gun accessible to his kid, it is foreseeable that one of his kids may misuse it thereby causing an injury. This question asks about conduct that is negligent because of the prospect of improper conduct by another person. The rule is that conduct of the defendant can lack reasonable care if it foreseeably combines with improper conduct of another. Restatement (Third) Torts § 19.

Alice was exposed to two different solvents while working in a laboratory. Each solvent contained a toxic chemical. One contained Chemical B; the other contained Chemical C. After developing the disease decolis several years later, Alice sues the manufacturers of each solvent, claiming that the manufacturers were negligent for including a toxic chemical in their solvents. Alice's evidence, presented by competent expert testimony based on valid scientific evidence reveals that the mechanism by which decolis develops is different for Chemical C exposure and for Chemical B exposure and that exposure to one or the other but not both is the most likely explanation for Alice's decolis. However, Alice cannot prove whether Chemical B or Chemical C caused her decolis. After presentation of her case, the defendants (manufacturers of Chemical B and manufacturer of Chemical C) both move for summary judgment on the issue of factual causation. How should the court rule on the defendants' requests for summary judgment?

The court should deny the motion for summary judgment and rule that the burden of proof on agent-disease factual causation is shifted to the manufacturers of Chemical B and Chemical C. Correct. This question raises the issue of proof problems in proving negligence when there are multiple actors. Usually, the plaintiff has the burden of proof for every element of the prima facie case, but when the plaintiff sues multiple tortfeasors and cannot reasonably be expected to prove which one caused her harm, the burden of proof on the issue of factual causation shifts to the defendants. Restatement (Third) Torts § 28.

Psychotherapist is counseling adult patient as patient is going through a divorce. After three or four sessions patient expresses a romantic interest in psychotherapist. Psychotherapist asks patient if patient is sure about how he feels. Patient responds that he is sure and psychotherapist and patient engage in a sexual relationship. Does the patient have a potential breach of fiduciary duty case against the psychotherapist?

Yes, because psychotherapists are in a position of power over their patients and as a result consent of the patient for a sexual relationship is not valid. Correct. Psychotherapists are deemed to have such a huge influence on the mental state of their patients that any consent by the patient for sexual relationship with the psychotherapist is deemed invalid.


संबंधित स्टडी सेट्स

Skin / Mobility Prep U Leukhardt

View Set

Unit 26-34: 'can'/'could'/'able to'/'should'/'must'/'have'/'need'

View Set

expected range of sugar levels mg/dL?

View Set

Chapter 12: Intra & Postpartum Periods (Test)

View Set

Lecture Chapter 3: Cellular Form and Function

View Set

strangers to these shores chapter 4

View Set

Chapter 1: Basic Concepts and Computer Evolution

View Set

Ch 13: Assignment - Investment Fundamentals

View Set

Ch 46: Assessment and Management of Patients with Diabetes

View Set

Guarantee ExamFX, Idaho Personal Lines Simulation Exam

View Set

Civil Procedure Master Federal Rules List

View Set